Question 15.7: Determine the reactions at the supports for the prismatic be...

Determine the reactions at the supports for the prismatic beam and loading shown in Fig. 15.29. (This is the same beam and loading as in Example 15.5 of Sec. 15.5.)

15.29
The blue check mark means that this solution has been answered and checked by an expert. This guarantees that the final answer is accurate.
Learn more on how we answer questions.

We consider the reaction at B as redundant and release the beam from the support. The reaction R_{B} is now considered as an unknown load (Fig. 15.30a) and will be determined from the condition that the deflection of the beam at B must be zero. The solution is carried out by considering separately the deflection (y_{B})_{w} caused at B by the uniformly distributed load ω (Fig. 15.30b) and the deflection (y_{B})_{R} produced at the same point by the redundant reaction R_{B} (Fig. 15.30c).

From the table of App. C (cases 2 and 1), we find that

(y_{B})_{w} = – \frac{wL^{4}}{8EI}                         (y_{B})_{R} = + \frac{R_{B}L^{3}}{3EI}

Writing that the deflection at B is the sum of these two quantities and that it must be zero, we have

y_{B} = (y_{B})_{w }+ (y_{B})_{R} = 0

y_{B} = – \frac{wL^{4}}{8EI} + \frac{R_{B}L^{3}}{3EI}= 0

and, solving for R_{B},             R_{B}= \frac{3}{8} wL                       R_{B} = \frac{3}{8}wL ↑

Drawing the free-body diagram of the beam (Fig. 15.31) and writing  the corresponding equilibrium equations, we have

+↑ \sum{F_{y}} = 0:      R_{A} + R_{B} – wL = 0                             (15.46)

R_{A} = wL – R_{B} = wL – \frac{3}{8} wL = \frac{5}{8} wL

R_{A} = \frac{5}{8} wL ↑

+\curvearrowleft \sum{M_{A}} = 0:      M_{A} + R_{B}L – (wL) (\frac{1}{2} L) = 0                                      (15.47)

M_{A} = \frac{1}{2} wL^{2} – R_{B}L = \frac{1}{2} wL^{2} – \frac{3}{8} wL^{2} = \frac{1}{8}wL^{2}

M_{A} = \frac{1}{8} wL^{2} \curvearrowleft

Alternative Solution. We may consider the couple exerted at the fixed end A as redundant and replace the fixed end by a pin-and-bracket support. The couple M_{A} is now considered as an unknown load (Fig. 15.32a) and will be determined from the condition that the slope of the beam at A must be zero. The solution is carried out by considering separately the slope (θ_{A})_{w} caused at A by the uniformity distributed load w (Fig. 15.32b) and the slope (θ_{A})_{M} produced at the same point by the unknown couple M_{A} (Fig. 15.32c).

Using the table of App. C (cases 6 and 7), and noting that in case 7, A and B must be interchanged, we find that

(θ_{A})_{w} = – \frac{wL^{3}}{24EI}               (θ_{A})_{M} = \frac{M_{A}L}{3EI}

Writing that the slope at A is the sum of these two quantities and that it must be zero, we have

θ_{A} = (θ_{A})_{w} + (θ_{A})_{M} = 0

θ_{A} =- \frac{wL^{3}}{25EI}+ \frac{M_{A}L}{3EI} = 0

and, solving for M_{A},

M_{A} = \frac{1}{8} wL^{2}                        M_{A} = \frac{1}{8} wL^{2} \curvearrowleft

The values of R_{A}   and   R_{B} may then be found from the equilibrium equations (15.46) and (15.47).

15.30
15.31
15.32

Related Answered Questions

Question: 15.S-P.6

Verified Answer:

Principle of Superposition. Assuming the axial for...
Question: 15.S-P.5

Verified Answer:

Principle of Superposition. The reaction R_...
Question: 15.S-P.4

Verified Answer:

Principle of Superposition. The given loading can ...
Question: 15.S-P.2

Verified Answer:

Differential Equation of the Elastic Curve. From E...
Question: 15.5

Verified Answer:

Equilibrium Equations. From the free-body diagram ...